www.vorhilfe.de
- Förderverein -
Der Förderverein.

Gemeinnütziger Verein zur Finanzierung des Projekts Vorhilfe.de.
Hallo Gast!einloggen | registrieren ]
Startseite · Mitglieder · Impressum
Forenbaum
^ Forenbaum
Status VH e.V.
  Status Vereinsforum

Gezeigt werden alle Foren bis zur Tiefe 2

Navigation
 Startseite...
 Suchen
 Impressum
Das Projekt
Server und Internetanbindung werden durch Spenden finanziert.
Organisiert wird das Projekt von unserem Koordinatorenteam.
Hunderte Mitglieder helfen ehrenamtlich in unseren moderierten Foren.
Anbieter der Seite ist der gemeinnützige Verein "Vorhilfe.de e.V.".
Partnerseiten
Weitere Fächer:

Open Source FunktionenplotterFunkyPlot: Kostenloser und quelloffener Funktionenplotter für Linux und andere Betriebssysteme
Forum "Uni-Analysis-Induktion" - Produktformel Beweis
Produktformel Beweis < Induktion < eindimensional < reell < Analysis < Hochschule < Mathe < Vorhilfe
Ansicht: [ geschachtelt ] | ^ Forum "Uni-Analysis-Induktion"  | ^^ Alle Foren  | ^ Forenbaum  | Materialien

Produktformel Beweis: Tipp zu Umformung
Status: (Frage) beantwortet Status 
Datum: 23:02 Do 24.10.2013
Autor: RoMalle

Ich habe diese Frage in keinem Forum auf anderen Internetseiten gestellt.


Hallo zusammen. Ich habe folgende Aufgabe bei der ich euren Rat brauche. Gegeben ist folgendes Produkt:

[mm] \produkt_{k=2}^{n}\bruch{k^2}{k^2 - 1} [/mm]

Für dieses Produkt soll ich nun eine möglichst einfache Formel herleiten und sie dann beweisen.
Es scheitert bei mir jedoch schon bei der Herleitung der einfachen Formel. Mein Ansatz war der, dass ich das Produkt für verschiedene Werte für n ausgeschrieben und nach Mustern ausschau gehalten hab, aus denen ich dann eine Formel, im Sinne von [mm] n*irgendwas^2, [/mm] hätte formulieren können. Es hat nicht geklappt. Wenn ich mal dachte ich hätte ne gute Formel, dann hat diese nur bei niedriegen n-Werten funktioniert und bei den großen versagt, oder anders rum. Am Ende will ich einfach irgendwas im Sinne von

[mm] \produkt_{k=2}^{n}\bruch{k^2}{k^2 - 1} [/mm] = [mm] \bruch{(n+1)^2 *2n}{n^2} [/mm] ????
da stehen haben.
Könntet ihr mir bitte einige Tipps geben? Ich brauche einfach nur die vereinfachte Formel, die vollständige Induktion würde ich dann schon selber hin kriegen.

Ich Danke euch im Vorraus

Gruß

Ro

        
Bezug
Produktformel Beweis: Werte einsetzen
Status: (Antwort) fertig Status 
Datum: 23:44 Do 24.10.2013
Autor: Loddar

Hallo RoMalle,

[willkommenmr] !!


Schreibe Dir einfach mal die ersten Glieder des Produktes auf.

$n \ = \ 2 \ : \ \ [mm] \bruch{2^2}{2^2-1} [/mm] \ = \ [mm] \bruch{4}{3}$ [/mm]

$n \ = \ 3 \ : \ \ [mm] \bruch{4}{3}*\bruch{3^2}{3^2-1} [/mm] \ = \ [mm] \bruch{4}{3}*\bruch{9}{8} [/mm] \ = \ [mm] \bruch{3}{2} \blue{\ = \ \bruch{6}{4}}$ [/mm]

$n \ = \ 4 \ : \ \ [mm] \bruch{3}{2}*\bruch{4^2}{4^2-1} [/mm] \ = \ [mm] \bruch{3}{2}*\bruch{16}{15} [/mm] \ = \ [mm] \bruch{8}{5}$ [/mm]

$n \ = \ 5 \ : \ \ [mm] \bruch{8}{5}*\bruch{53^2}{5^2-1} [/mm] \ = \ [mm] \bruch{8}{5}*\bruch{25}{24} [/mm] \ = \ [mm] \bruch{5}{3} \blue{\ = \ \bruch{10}{6}}$ [/mm]

$n \ = \ 6 \ : \ \ [mm] \bruch{5}{3}*\bruch{6^2}{6^2-1} [/mm] \ = \ [mm] \bruch{5}{3}*\bruch{36}{35} [/mm] \ = \ [mm] \bruch{12}{7}$ [/mm]

$n \ = \ 7 \ : \ \ [mm] \bruch{12}{7}*\bruch{7^2}{7^2-1} [/mm] \ = \ [mm] \bruch{12}{7}*\bruch{49}{48} [/mm] \ = \ [mm] \bruch{7}{4} \blue{\ = \ \bruch{14}{8}}$ [/mm]

Hier sollte sich eine Gesetzmäßigkeit doch eindeutig erkennen lassen.


Gruß
Loddar

Bezug
        
Bezug
Produktformel Beweis: Antwort
Status: (Antwort) fertig Status 
Datum: 01:33 Fr 25.10.2013
Autor: HJKweseleit

Zähler und Nenner sind sehr ähnlich, deshalb versuchst du, durch Kürzen Faktoren zu verrechnen.


[mm]\produkt_{k=2}^{n}\bruch{k^2}{k^2 - 1}=\produkt_{k=2}^{n}\bruch{k^2}{(k - 1)*(k+1)} =\bruch{\produkt_{k=2}^{n}k*\produkt_{k=2}^{n}k}{\produkt_{k=2}^{n}(k - 1)*\produkt_{k=2}^{n}(k+1)}[/mm]

Jetzt wird so umindiziert, dass alles gleich aussieht, nämlich

... = [mm] \bruch{\produkt_{k=2}^{n}k*\produkt_{k=2}^{n}k}{\produkt_{k=1}^{n-1}k*\produkt_{k=3}^{n+1}k}. [/mm]

Nun kürzen sich im "Kern" (von k=3 bis k=n-1) alle Faktoren weg bis auf die überzähligen Randwerte (k=1, k=2, k=n, k=n+1):

...  [mm] =\bruch{2*n*2*n}{1*2*n*(n+1)} =\bruch{2*n}{(n+1)} [/mm]


Bezug
        
Bezug
Produktformel Beweis: Antwort
Status: (Antwort) fertig Status 
Datum: 02:25 Fr 25.10.2013
Autor: Marcel

Hallo,

> Ich habe diese Frage in keinem Forum auf anderen
> Internetseiten gestellt.
>  
>
> Hallo zusammen. Ich habe folgende Aufgabe bei der ich euren
> Rat brauche. Gegeben ist folgendes Produkt:
>  
> [mm]\produkt_{k=2}^{n}\bruch{k^2}{k^2 - 1}[/mm]

eine "gern benutzte Standardidee", wenn man sich mit Produkten noch
nicht so auskennt, ist es, mal den [mm] $\ln$ [/mm] draufzuschmeißen (beachte, dass
Dein Produkt sicher [mm] $>0\,$ [/mm] ist):

    [mm] $\ln\left(\produkt_{k=2}^n \frac{k^2}{k^2-1}\right)=\sum_{k=2}^n (\ln(k^2)-\ln(k^2-1))=2*\sum_{k=2}^n \ln(k)\;-\;\sum_{k=2}^n \ln((k+1)(k-1))=2*\sum_{k=2}^n \ln(k)\;-\;\sum_{k=2}^n \ln(k+1)\;-\;\sum_{k=2}^n \ln(k-1))$ [/mm]

Ist aber eigentlich i.W. das gleiche Vorgehen, wie HJKweseleit vorgeschlagen
hat und von daher macht man hier eigentlich etwas unnötig, indem man
den [mm] $\ln$ [/mm] benutzt. Nichtsdestotrotz kommen manche einfach besser mit
dem Summenzeichen zurecht (warum auch immer - im Endeffekt ist das
ja auch nur "ein Verknüpfungszeichen"). Von daher auch mal dieser Vorschlag!

P.S.
Beachte: [mm] $\ln(x)=y$ [/mm] für $x > [mm] 0\,$ [/mm] ist gleichwertig mit [mm] $x=\exp(y)\,.$ [/mm] Wenn Du also oben

     [mm] $\ln\left(\produkt_{k=2}^n \frac{k^2}{k^2-1}\right)=\text{Formel}(n)$ [/mm]

hast, dann hast Du auch [mm] $\produkt_{k=2}^n \frac{k^2}{k^2-1}=\exp(\text{Formel}(n))\,.$ [/mm]

Gruß,
  Marcel

Bezug
        
Bezug
Produktformel Beweis: Frage (beantwortet)
Status: (Frage) beantwortet Status 
Datum: 10:08 Fr 25.10.2013
Autor: RoMalle

@ Loddar, auf die Idee die Ergebnisse auf die bei dir in blau dargestellten Werte zu formen bin ich gar nicht gekommen. Manchmal sieht man den Wald vor lauter Bäumen nicht. Danke deine Idee hat meine Aufgabe gelöst. ;-)

@ HJKweseleit. Dir ebenfalls ein Danke für deinen Lösungsvorschlag, der mir übrigens auch sehr gut gefallen hat, da dieser sehr elegant ist. Aber eine Frage hätte ich da noch.

Wie man von [mm] \produkt_{k=2}^{n} [/mm] auf 2n kommt scheint mir trivial. Doch mir verschliessen sich die Umformungsschritte von [mm] \produkt_{k=1}^{n-1}*\produkt_{k=3}^{n+1} [/mm] zu 1*2*n*(n+1). Könntest du mir da bitte noch einpaar genauere Erklärungen für geben?

@ Marcel. Dir will ich auch für deine Antwort danken. Ich tue mich durchaus schwerer mit dem Produktzeichen als mit dem Summenzeichen. In der Vorlesungen haben wir Ersteres nicht wirklich durch genommen, der Prof hat nur nebenbei mal erwähnt, dass so was auch in der Mathe Welt vorkommt. Aufjedenfall werde ich deine Idee mit mit ln im Hinterkopf behalten.

Bezug
                
Bezug
Produktformel Beweis: Antwort
Status: (Antwort) fertig Status 
Datum: 12:09 Fr 25.10.2013
Autor: Marcel

Hallo,

ertmal vorneweg: Die "einfachste Erklärung" findet sich ganz am Ende.
Tatsächlich ist hier aber die "einfachste Erklärung" in Wahrheit eher die
Erklärung, die am meisten verschleiert, was hier zum Tragen kommt.
Daher wundere Dich nicht, dass ich das Ganze erstmal in einer
vorhergehenden Erklärung "abstrahiere". (Du kannst aber auch sagen:
Okay, fange ich doch erstmal "mit der einfachsten Erklärung" an. Dann
scrolle einfach nach unten, bis Du die beiden Zeilen mit gestrichelten
Linien siehst - das darauffolgende ist "elementare Erklärung", wenn man
es so nennen will...)

> @ Loddar, auf die Idee die Ergebnisse auf die bei dir in
> blau dargestellten Werte zu formen bin ich gar nicht
> gekommen. Manchmal sieht man den Wald vor lauter Bäumen
> nicht. Danke deine Idee hat meine Aufgabe gelöst. ;-)
>  
> @ HJKweseleit. Dir ebenfalls ein Danke für deinen
> Lösungsvorschlag, der mir übrigens auch sehr gut gefallen
> hat, da dieser sehr elegant ist. Aber eine Frage hätte ich
> da noch.
>  
> Wie man von [mm]\produkt_{k=2}^{n}[/mm] auf 2n kommt scheint mir
> trivial. Doch mir verschliessen sich die Umformungsschritte
> von [mm]\produkt_{k=1}^{n-1}*\produkt_{k=3}^{n+1}[/mm]Eingabefehler: "\left" und "\right" müssen immer paarweise auftreten, es wurde aber ein Teil ohne Entsprechung gefunden (siehe rote Markierung)

zu

> 1*2*n*(n+1). Könntest du mir da bitte noch einpaar
> genauere Erklärungen für geben?

Du hast alles verstanden bis zur Stelle

    $\bruch{\red{\produkt_{k=2}^{n}k}\cdot{}\produkt_{k=2}^{n}k}{\blue{\produkt_{k=1}^{n-1}k}\cdot{}\green{\produkt_{k=3}^{n+1}k}},$
oder?

Bedenke nun mal, dass folgende Dinge gelten

    $\red{\produkt_{k=2}^{n}k}=\left(\blue{\produkt_{k=2}^{n-1}k}\right)*n\,,$

    $\blue{\produkt_{k=\textbf{2}}^{n-1}k}=\blue{\produkt_{k=\red{2}}^{n-1}k}\right)\,.$

Mache Dir damit klar, dass man mit Kürzung erreicht

    $\bruch{\red{\produkt_{k=2}^{n}k}}{\blue{\produkt_{k=1}^{n-1}k}}}=n\,.$

Damit haben wir, wenn wir weitermachen

    $\bruch{\red{\produkt_{k=2}^{n}k}\cdot{}\produkt_{k=2}^{n}k}{\blue{\produkt_{k=1}^{n-1}k}\cdot{}\green{\produkt_{k=3}^{n+1}k}}=\frac{\red{\produkt_{k=2}^{n}k}}{\blue{\produkt_{k=1}^{n-1}k}}*\frac{\blue{\produkt_{k=1}^{n-1}k}}{\green{\produkt_{k=3}^{n+1}k}}}=n*\frac{\blue{\produkt_{k=1}^{n-1}k}}{\green{\produkt_{k=3}^{n+1}k}}}\,.$

Jetzt versuche mal, den verbleibenden Faktor (Bruch) ähnlich zu analysieren.

P.S. Du kennst sicher analoges:

    $\sum_{k=1}^{n-1}a_k-\sum_{k=3}^{n+1}a_k=(a_1+a_2)+\sum_{k=3}^{n-1}a_k-\left\{\left(\sum_{k=3}^{n-1}a_k\right)+(a_n+a_{n+1})\right\}=(a_1+a_2)-(a_n+a_{n+1})$  ($=a_1+a_2-a_n-a_{n+1}$).

Und manchmal fällt der Groschen am Besten, wenn man es allgemein
notiert (abstrakte Erklärung):
Sei $(G,\circ)$ eine abelsche Gruppe mit neutralem Element $e\,$ und Verknüpfung
$\circ$. Wir schreiben $\odot\limits_{k=1}^n g_k:=g_1 \circ g_2 \circ ... \circ g_n$ (warum ist das eine unproblematische Definition?).
Weiter schreiben wir, wie üblich, wenn $g \in G$ vorgegeben ist, für das (eindeutig
bestimmte) Element $a \in G$ mit $a \circ g=e$ kurz $g^{-1}:=a\,.$

Dann gilt etwa

    $\left(\odot_{k=2}^n g_k\right)\circ \left(\odot_{k=3}^{n+1}g_k\right)^{-1}=\left(\odot_{k=2}^n g_k\right) \circ (\left(\odot_{k=3}^{n+1} {(g_k)}^{-1}\right))=...=g_2 \circ {(g_{n+1})}^{-1}\,.$      

Bei der Rechnung wird zweimal benutzt, dass die Gruppe abelsch ist, denn
in allgemeinen Gruppen gilt schon nur $(a \circ b)^{-1}=b^{-1} \circ a^{-1}\,.$
Und bei den "..." kommt die "Aufspaltung" zu tragen, wenn  man es ausführlich
hinschreibt (also die Assoziativität).

----------------------------------------------------------------
----------------------------------------------------------------

Aber, bevor wir uns hier zu sehr "im Abstrakten" verlieren (denn manche
haben wiederum damit mehr Probleme):
Du kannst auch mal "elementar" einfach folgendes machen - nämlich einfach
die Produkte "ausschreiben":

    $\bruch{\red{\produkt_{k=2}^{n}k}\cdot{}\produkt_{k=2}^{n}k}{\blue{\produkt_{k=1}^{n-1}k}\cdot{}\green{\produkt_{k=3}^{n+1}k}}=\frac{\red{(2*3*...*n)}*\red{(2*3*...*n)}}{\blue{(1*2*3*...*(n-1))*\green{(3*4*...*n*(n+1))}}}=\underbrace{\frac{\red{(2*3*...*n)}}{\blue{(1*2*...*(n-1))}}}_{=n \text{ (Beachte: Kürzen!)}}*\frac{\red{(2*3*...*n)}}{\green{(3*4*...*n*(n+1))}}$

Das wäre die geeigneteste Methode, um Schülern das Ganze klar zu
machen, denke ich. Sie hat aber den Nachteil, dass eigentlich nur der
Wissende wirklich sieht, welche "Rechengesetze" da tatsächlich ihre
Anwendung finden.  

> @ Marcel. Dir will ich auch für deine Antwort danken. Ich
> tue mich durchaus schwerer mit dem Produktzeichen als mit
> dem Summenzeichen. In der Vorlesungen haben wir Ersteres
> nicht wirklich durch genommen, der Prof hat nur nebenbei
> mal erwähnt, dass so was auch in der Mathe Welt vorkommt.
> Aufjedenfall werde ich deine Idee mit mit ln im Hinterkopf
> behalten.

Wie gesagt: Im Endeffekt "baut man nur die Anischten" um - denn das
Prinzip ist das Gleiche und erklärt sich wie oben.

Gruß,
  Marcel

Bezug
        
Bezug
Produktformel Beweis: Antwort
Status: (Antwort) fertig Status 
Datum: 10:55 Fr 25.10.2013
Autor: abakus


> Ich habe diese Frage in keinem Forum auf anderen
> Internetseiten gestellt.

>
>

> Hallo zusammen. Ich habe folgende Aufgabe bei der ich euren
> Rat brauche. Gegeben ist folgendes Produkt:

>

> [mm]\produkt_{k=2}^{n}\bruch{k^2}{k^2 - 1}[/mm]

>

> Für dieses Produkt soll ich nun eine möglichst einfache
> Formel herleiten und sie dann beweisen.

Hallo,
hast du schon mal (im Nenner) an die dritte binomische Formel gedacht?
Das Produkt ist AUCH
[mm] \frac{2*2}{3*1}* \frac{3*3}{4*2}* \frac{4*4}{5*3}* \frac{5*5}{6*4}* \frac{...}{...}[/mm]

Bis zu dem Bruch mit ... kürzen sich weg:
-alle Faktoren 3
-alle Faktoren 4
-ein Faktor 2
-ein Faktor 5

Übrig bleiben:
- im Zähler eine 2 und eine 5 (= erstes k und bis dahin letztes k)
- im Nenner eine 1 und eine 6 (= Vorgänger der ersten k und Nachfolger des bis dahin letzten k).
Wenn k bis n läuft, lautet der Zähler also 2*n
und der Nenner 1*(n+1).

Gruß Abakus


> Es scheitert bei mir jedoch schon bei der Herleitung der
> einfachen Formel. Mein Ansatz war der, dass ich das Produkt
> für verschiedene Werte für n ausgeschrieben und nach
> Mustern ausschau gehalten hab, aus denen ich dann eine
> Formel, im Sinne von [mm]n*irgendwas^2,[/mm] hätte formulieren
> können. Es hat nicht geklappt. Wenn ich mal dachte ich
> hätte ne gute Formel, dann hat diese nur bei niedriegen
> n-Werten funktioniert und bei den großen versagt, oder
> anders rum. Am Ende will ich einfach irgendwas im Sinne
> von

>

> [mm]\produkt_{k=2}^{n}\bruch{k^2}{k^2 - 1}[/mm] = [mm]\bruch{(n+1)^2 *2n}{n^2}[/mm]
> ????
> da stehen haben.
> Könntet ihr mir bitte einige Tipps geben? Ich brauche
> einfach nur die vereinfachte Formel, die vollständige
> Induktion würde ich dann schon selber hin kriegen.

>

> Ich Danke euch im Vorraus

>

> Gruß

>

> Ro

Bezug
Ansicht: [ geschachtelt ] | ^ Forum "Uni-Analysis-Induktion"  | ^^ Alle Foren  | ^ Forenbaum  | Materialien


^ Seitenanfang ^
ev.vorhilfe.de
[ Startseite | Mitglieder | Impressum ]